Tải bản đầy đủ (.pdf) (73 trang)

Bất đẳng thức và bài toán cực trị trong lớp các đa thức và phân thức hệ số nguyên (LV thạc sĩ)

Bạn đang xem bản rút gọn của tài liệu. Xem và tải ngay bản đầy đủ của tài liệu tại đây (426 KB, 73 trang )

ĐẠI HỌC THÁI NGUYÊN
TRƯỜNG ĐẠI HỌC KHOA HỌC

NGUYỄN NGỌC HÀ

BẤT ĐẲNG THỨC VÀ BÀI TOÁN CỰC
TRỊ TRONG LỚP CÁC ĐA THỨC VÀ
PHÂN THỨC HỆ SỐ NGUYÊN

LUẬN VĂN THẠC SỸ TOÁN HỌC

THÁI NGUYÊN - NĂM 2016


ĐẠI HỌC THÁI NGUYÊN
TRƯỜNG ĐẠI HỌC KHOA HỌC

NGUYỄN NGỌC HÀ

BẤT ĐẲNG THỨC VÀ BÀI TOÁN CỰC
TRỊ TRONG LỚP CÁC ĐA THỨC VÀ
PHÂN THỨC HỆ SỐ NGUYÊN

LUẬN VĂN THẠC SỸ TOÁN HỌC

Chuyên ngành: PHƯƠNG PHÁP TOÁN SƠ CẤP
Mã số: 60 46 01 13

Người hướng dẫn khoa học:
GS. TSKH. NGUYỄN VĂN MẬU


THÁI NGUYÊN - NĂM 2016


i

Mục lục
Mở đầu

1

1 Một số kiến thức chuẩn bị

3

1.1

Một số tính chất cơ bản của đa thức với hệ số nguyên . . . .

3

1.2

Một số tính chất của phân thức hữu tỉ với hệ số nguyên . . .

8

1.3

Định lý Viète . . . . . . . . . . . . . . . . . . . . . . . . . .


12

1.4

Một số bất đẳng thức cơ bản . . . . . . . . . . . . . . . . . .

13

2 Các dạng toán về đa thức và phân thức với hệ số nguyên
2.1

19

Các dạng toán về đa thức một biến với hệ số nguyên và hệ
thức Viète . . . . . . . . . . . . . . . . . . . . . . . . . . . .

19

2.2

Đa thức với các hệ số nguyên và đồng dư thức . . . . . . . .

30

2.3

Bất đẳng thức phân thức sinh bởi tam thức bậc hai trên một
khoảng . . . . . . . . . . . . . . . . . . . . . . . . . . . . . .

35


2.4

Bất đẳng thức sinh bởi hàm phân tuyến tính trên một khoảng

40

2.5

Phân thức chính quy và một số tính chất . . . . . . . . . . .

45

3 Một số bài toán liên quan đến bất đẳng thức và cực trị trên
tập số nguyên

49

3.1

Bất đẳng thức trên tập số nguyên . . . . . . . . . . . . . . .

49

3.2

Cực trị trên tập số nguyên . . . . . . . . . . . . . . . . . . .

58


Kết luận

69

Tài liệu tham khảo

70


1

Mở đầu
Chuyên đề đa thức là một chuyên đề rất quan trọng ở bậc trung học phổ
thông. Đa thức không chỉ là đối tượng nghiên cứu trọng tâm của Đại số mà
còn là công cụ đắc lực trong nhiều lĩnh vực khác của toán học.
Trong các kì thi học sinh giỏi toán các cấp, Olympic Toán sinh viên, các
bài toán liên quan tới đa thức nói chung và đặc biệt là các bài toán về bất
đẳng thức, cực trị của đa thức, phân thức có hệ số nguyên thường xuyên
được đề cập. Những dạng toán này thường được xem là thuộc loại khó, hơn
nữa phần kiến thức về đa thức, phân thức hệ số nguyên lại không nằm trong
chương trình chính thức của Số học và Đại số bậc trung học phổ thông.
Để đáp ứng nhu cầu bồi dưỡng giáo viên và bồi dưỡng học sinh giỏi về
chuyên đề đa thức, tôi đã làm luận văn: Bất đẳng thức và bài toán cực trị
trong lớp các đa thức và phân thức hệ số nguyên. Luận văn gồm phần mở
đầu, ba chương, phần kết luận và danh mục tài liệu tham khảo.
Chương I trình bày các kiến thức cơ bản về đa thức và phân thức hệ số
nguyên, định lý Viète, một số bất đẳng thức cơ bản.
Chương II trình bày một số dạng toán về bất đẳng thức và cực trị trong
lớp các đa thức và phân thức hệ số nguyên, phân thức chính quy và áp dụng.
Chương III trình bày một số bất đẳng thức và bài toán cực trị trên tập

số nguyên.
Luận văn có thể được xem như một tài liệu bồi dưỡng giáo viên và bồi
dưỡng học sinh giỏi về chuyên đề đa thức. Có thể sử dụng luận văn trong
việc giảng dạy học sinh thi học sinh giỏi các cấp, Olympic sinh viên.


2

Luận văn được hoàn thành dưới sự hướng dẫn trực tiếp của GS.TSKH
Nguyễn Văn Mậu. Tác giả xin bày tỏ lòng biết ơn sâu sắc về sự chỉ bảo tận
tình của Thầy trong suốt quá trình xây dựng đề cương cũng như hoàn thành
luận văn.
Tác giả xin gửi lời cảm ơn chân thành tới TS Nguyễn Thị Thu Thủy và
các quí thầy cô đã đọc, kiểm tra, đánh giá và đưa ra những ý kiến quý báu
để luận văn được hoàn thiện hơn.
Tác giả xin chân thành cảm ơn quý Thầy Cô trong Ban Giám hiệu, phòng
sau Đại học, khoa Toán Tin trường Đại học Khoa học, Đại học Thái Nguyên
tạo điều kiện thuận lợi trong suốt quá trình học tập và hoàn thành luận văn.
Trong khuôn khổ một luận văn, tác giả chưa thể trình bày được hết các
vấn đề về đa thức và phân thức hệ số nguyên. Tuy bản thân đã có nhiều
cố gắng, nỗ lực nghiên cứu, song do điều kiện và trình độ còn hạn chế nên
những kết quả đạt được trong luận văn còn rất khiêm tốn. Tác giả kính mong
nhận được ý kiến đóng góp quí báu của các thầy cô để bản luận văn được
hoàn thiện hơn.
Tác giả xin chân thành cảm ơn!

Thái Nguyên, ngày 20 tháng 5 năm 2016.
Học viên

Nguyễn Ngọc Hà



3

Chương 1
Một số kiến thức chuẩn bị
Trong chương này sẽ trình bày một số kiến thức cơ bản: định nghĩa, một
số tính chất về sự chia hết, về nghiệm nguyên, về hệ số . . . của đa thức và
phân thức hệ số nguyên. Ở đây ta sẽ sử dụng một số ký hiệu: Cho đa thức

f (x) = an xn + an−1 xn−1 + · · · + a1 x + a0 , nếu f (x) có các hệ số là số nguyên
thì ta ký hiệu f (x) ∈ Z[x], nếu f (x) có các hệ số là số hữu tỉ thì ta ký hiệu

f (x) ∈ Q[x].

1.1

Một số tính chất cơ bản của đa thức với hệ số
nguyên

Định lý 1.1 (xem [4]). Cho đa thức f (x) = an xn +an−1 xn−1 +· · ·+a1 x+a0 ∈
.
Z[x], an = 0, a là số nguyên. Khi đó [f (x) − f (a)]..(x − a).
Chứng minh.
Ta có

f (a) = an an + an−1 an−1 + · · · + a1 a + a0 ,
.
f (x) − f (a) = an (xn − an ) + an−1 xn−1 − an−1 + · · · + a1 (x − a)..(x − a).
Vậy ta có điều cần chứng minh.



4

p
Bài toán 1.1 (xem [4]). Chứng minh rằng nếu phân số tối giản , ((p, q) =
q
1) là nghiệm của đa thức với hệ số nguyên
f (x) = an xn + an−1 xn−1 + · · · + a1 x + a0
thì p là ước của a0 và q là ước của an .
Lời giải.
Giả sử phân thức tối giản

f

p
q

p
q

= an

p
là nghiệm của đa thức f(x). Khi đó, ta có
q

n

+ an−1


p
q

n−1

+ · · · + a1

p
q

+ a0 = 0.

Từ đó, ta có

an pn = −q(an−1 pn−1 + · · · + a1 q n−2 p + a0 q n−1 )

(1.1)

a0 q n = −p(an pn−1 + an−1 pn−2 q + · · · + a1 q n−1 ).

(1.2)



Từ (1.1) suy ra an pn chia hết cho q mà (p, q) = 1 nên an chia hết cho q .
Từ (1.2)suy ra a0 q n chia hết cho p mà (p, q) = 1 nên a0 chia hết cho p.

p
Bài toán 1.2 (xem [4]). Chứng minh rằng nếu phân thức tối giản , ((p, q) =

q
1) là nghiệm của đa thức với hệ số nguyên
f (x) = an xn + an−1 xn−1 + · · · + a1 x + a0
thì p − mq là ước của f (m) với m là số nguyên.
Lời giải.
Phân tích f (x) theo các lũy thừa của (x − m) ta được

f (x) = an (x − m)n + bn−1 (x − m)n−1 + · · · + b1 (x − m) + b0 = g(x − m).
Nhận xét rằng các hệ số b0 , bn−1 là các số nguyên vì m là một số nguyên.
p
Ta có f (m) = b0 . Thay x bởi ta thu được đẳng thức
q

f

p
q

=g

p
−m =g
q

p − mq
q

= 0.



5

p − mq
là nghiệm của g(x). theo Bài toán 2.6 thì p − mq là ước của
q
b0 = f (m).
Do đó

Bài toán 1.3. Cho đa thức f (x) có hệ số nguyên thỏa mãn điều kiện

f (0), f (1), . . . , f (m − 1) đều không chia hết cho m (m là số nguyên dương
cho trước, m > 1). Chứng minh rằng f (x) = 0 không có nghiệm nguyên.
Lời giải.
Giả sử f (x) = 0 có nghiệm nguyên là x = c, khi đó

f (x) = (x − c)g(x), g(x) ∈ Z[x].
Ta có

f (0) = (0 − c)g(0),
f (1) = (1 − c)g(1),
...
f (m − 1) = (m − 1 − c)g(m − 1).
Vì 0 − c, 1 − c, . . . , m − 1 − c là m số nguyên liên tiếp nên phải có một số
chia hết cho m.
Vì vậy trong m số f (0), f (1), . . . , f (m − 1) phải có ít nhất một số chia
hết cho m. Điều này trái với giả thiết.
Vậy f (x) = 0 không có nghiệm nguyên.
Bài toán 1.4. Cho đa thức P (x) với các hệ số nguyên, chia hết cho 3 khi x
.
lấy các giá trị nguyên k, k + 1, k + 2. Chứng minh rằng P (m) ..3 với mọi số

nguyên m.
Chứng minh.
.
Với hai số nguyên m và n phân biệt, ta có P (m) − P (n)..(m − n).
Ta có các số P (m) − P (k), P (m) − P (k + 1) và P (m) − P (k + 2) theo
thứ tự đó lần lượt chia hết cho m − k, m − (k + 1), m − (k + 2) với mọi

m∈
/ {k, k + 1, k + 2}.


6

Vì m − k, m − (k + 1), m − (k + 2) là ba số nguyên liên tiếp nên trong đó
có một số chia hết cho 3. Do đó trong các số P (m) − P (k), P (m) − P (k + 1)
và P (m) − P (k + 2) có một số chia hết cho 3.
Mặt khác, theo giả thiết, các số P (k), P (k + 1), P (k + 2) đều chia hết cho
.
3. Vậy P (m)..3 với mọi số nguyên m.
Bài toán 1.5 (xem [4]). Cho đa thức f (x) ∈ Z[x]. Chứng minh rằng nếu
phương trình f (x) = 1 có nhiều hơn 3 nghiệm nguyên phân biệt thì phương
trình f (x) = −1 không có nghiệm nguyên.
Lời giải.
Giả sử phương trình f (x) = −1 có nghiệm nguyên là a thì f (a) = −1.
Gọi x1 , x2 , x3 , x4 là 4 nghiệm nguyên phân biệt của phương trình f (x) = 1,
thì

f (x) − 1 = (x − x1 )(x − x2 )(x − x3 )(x − x4 )g(x).
Suy ra


f (a) − 1 = −2 = (a − x1 )(a − x2 )(a − x3 )(a − x4 )g(a),
trong đó (a − x1 ), (a − x2 ), (a − x3 ), (a − x4 ) là 4 số nguyên phân biệt.
Nhưng −2 không thể phân tích được thành tích của 4 số nguyên khác
nhau nên điều giả sử ở trên là sai. Vậy phương trình f (x) = −1 không có
nghiệm nguyên.
Bài toán 1.6. Giả sử P (x) là đa thức bậc 1991 với hệ số nguyên. Xét đa
thức Q(x) = P 2 (x) − 9. Chứng minh rằng số nghiệm nguyên của đa thức

Q(x) nhỏ hơn 1996.
Lời giải.
Giả sử số nghiệm của đa thức Q(x) không nhỏ hơn 1996.

Q(x) = 0 ⇔ P 2 (x) − 9 = 0 ⇔ [P (x) − 3][P (x) + 3] = 0.


7

Gọi x1 , x2 , . . . , xk là các nghiệm nguyên của

P (x) = 3, (x1 < x2 < · · · < xk )
và y1 , y2 , . . . , yl là các nghiệm nguyên của

P (x) = −3, (y1 < y2 < · · · < yl ).
Rõ ràng xi = yj , ∀i, j . Vì deg P (x) = 1991 nên k ≤ 1991; l ≤ 1991.
Mặt khác k + l chính là số nghiệm của đa thức Q(x) nên theo giả thiết
phản chứng thì k + l ≥ 1996. Từ đó ta có k ≥ 5, l ≥ 5, suy ra tồn tại

i0 , j0 (1 ≤ i0 ≤ k; 1 ≤ j0 ≤ l) sao cho
|xi0 − yj0 | ≥ 7.


(1.3)

Giả sử

P (x) = a1991 x1991 + a1990 x1990 + · · · + a1 x + a0
với ai ∈ Z, i = 0, 1991.
Thế thì từ ai ∈ Z, i = 0, 1991, P (xi0 ) = 3 và P (yi0 ) = −3, suy ra

P (xi0 ) − P (yi0 ) = 6.
Vì P (x) là đa thức với hệ số nguyên và xi0 , yi0 là các số nguyên nên ta có
.
P (yi0 ) − P (xi0 )..yi0 − xi0 .
.
Như vậy 6..(yi0 − xi0 ), suy ra

yi0 − xi0 ≤ 6.

(1.4)

Từ (1.3) và (1.4) suy ra mâu thuẫn. Vậy giả thiết phản chứng là sai, tức là
đa thức Q(x) = P 2 (x) − 9 không thể có quá 1995 nghiệm nguyên.
Nhận xét 1.1. Ta có thể chứng minh (1.3) như sau: Vì k ≥ 5, l ≥ 5, xi =

yj , ∀i, j do đó có ít nhất ba nghiệm trong số các nghiệm yj (j = 1, l) nhỏ
hơn (hoặc lớn hơn) các nghiệm xi (i = 1, k). Giả sử

x1 < x2 < · · · < xk < yp < yp+1 < yp+2


8


(ở đây yp , yp+1 , yp+2 là 3 trong số các nghiệm y1 , y2 , y3 , . . . , yl ).
Chú ý là ở đây các xi (i = 1, k), yp , yp+1 , yp+2 đều là số nguyên nên

|yp+2 − x1 | = yp+2 − x1 ≥ k − 1 + 3 = k + 2.
Do k ≥ 5 nên suy ra |yp+2 − xi | ≥ 7.

1.2

Một số tính chất của phân thức hữu tỉ với hệ số
nguyên

Định nghĩa 1.1 (xem [4]). Hàm số f : R → R có dạng

f (x) =

P (x)
Q(x)

được gọi là phân thức hữu tỉ, trong đó P (x), Q(x) là các đa thức.
Nếu đa thức P (x) và Q(x) là các đa thức có hệ số hữu tỉ thì bằng việc
quy đồng mẫu số ta sẽ đưa f (x) về dạng

f (x) =

P1 (x)
Q1 (x)

trong đó P1 (x) và Q1 (x) là các đa thức có hệ số nguyên.
P (x)

Do vậy phân thức hữu tỉ f (x) =
được gọi là phân thức hữu tỉ có
Q(x)
hệ số nguyên nếu như P (x), Q(x) ∈ Q[x].
Bài toán 1.7 (xem [4]). Cho phân thức hữu tỉ

f (x) =

1
∈Q
ax + b

với mọi x ∈ Z. Chứng minh rằng a, b ∈ Q.
Chứng minh.
1
1
f (x) =
∈ Q với mọi x ∈ Z nên ax + b =
∈ Q với mọi x ∈ Z.
ax + b
f (x)
Vậy ax + b ∈ Q[x] hay a, b ∈ Q.


9

Bài toán 1.8 (xem [4]). Chứng minh rằng nếu f (x) =

x ∈ Z thì f (x) có dạng f (x) =


1
∈ Q với mọi
ax + b

C
với A, B, C thuộc Z.
Ax + B

Chứng minh.
1
f (x) =
∈ Q với mọi x ∈ Z nên theo Bài toán 1.7 ta có a, b ∈ Q.
ax + b
e
m
(m, n, e, f ∈ Z). Khi đó
Đặt a = , b =
n
f

nf
C
1
=
=
f (x) = m
e
mf x + ne Ax + B
x+
n

f
với A, B, C thuộc Z.
Bài toán 1.9 (xem [4]). Cho phân thức hữu tỉ f (x) =

ax + b
∈ Q với mọi
cx + d

x ∈ Z.
Chứng minh rằng f (x) có thể biểu diễn dưới dạng

f (x) =

Ax + B
(A, B, C, D ∈ Z).
Cx + D

(1.5)

Chứng minh.
Nếu ad − bc = 0 thì f (x) = const nên biểu diễn (1.5) là hiển nhiên.
Xét trường hợp ad − bc = 0.
Nếu c = 0 thì biểu diễn (1.5) là hiển nhiên.
f (x) − f (0)
1
Nếu c = 0 thì sử dụng phân tích
=
∈ Q với mọi
x
αx + β

x ∈ Z. Áp dụng Bài toán 1.8 ta sẽ được dạng biểu diễn (1.5).
Nhận xét rằng kết quả của Bài toán 1.9 cũng đúng trong trường hợp tổng
quát.
Bài toán 1.10 (xem [4]). Cho phân thức hữu tỉ

f (x) =

P (x)
∈ Q ∀x ∈ Z, (P (x), Q(x)) = 1.
Q(x)

Chứng minh rằng f (x) có thể biểu diễn được dưới dạng phân thức của hai
đa thức với hệ số nguyên.


10

Chứng minh.
Giả sử

P (x) = a0 + a1 x + · · · + am xm ,
Q(x) = b0 + b1 x + · · · + bn xn .
Tại x = j (j = 0, 1, . . . , m + n) hàm f (x) nhận các giá trị hữu tỉ tương
ứng là cj . Khi đó ta có hệ phương trình tuyến tính với m + n + 2 ẩn:

a0 , a1 , . . . , am , b0 , b1 , . . . , bn dạng
a0 + a1 k + · · · + am k m − b0 ck − b1 ck k − · · · − bn ck k n = 0,
trong đó k = 0, 1, . . . , m + n.
Hai nghiệm của hệ này cho ta hai cặp đa thức P (x), Q(x) và P1 (x), Q1 (x)
có tính chất


P (k) − ck Q(k) = 0, P1 (k) − ck Q1 (k) = 0, ∀k = 0, m + n.
Hai cặp nghiệm này cho ta đa thức

g(x) = P (x)Q1 (x) − P1 (x)Q(x), deg g(x) ≤ m + n
nhận giá trị 0 tại m + n + 1 điểm nên g(x) ≡ 0.
Do P (x) và Q(x) nguyên tố cùng nhau nên P (x) = cP1 (x); Q(x) =

cQ1 (x).
Vậy hệ đã cho chỉ có một nghiệm với sự sai khác một thừa tỉ lệ và như
vậy tồn tại ma trận cấp m + n + 1 trong ma trận hệ số của hệ phương trình
để định thức của nó khác 0 và nghiệm đã nhận được là các số hữu tỉ. Đây là
điều cần chứng minh.
Bài toán 1.11 (xem [4]). Cho p là một số nguyên dương, q ∈ [0, 1]. Giả sử

x ∈ [q p+1 , 1] và
p

x − qk
f (x) =
.
k
x
+
q
k=1


11


Chứng minh rằng

p

1 − qk
.
|f (x)| ≤
k
1
+
q
k=1
Chứng minh.
Ta có 0 < q p+1 < q p < · · · < q < 1. Với q j+1 ≤ x ≤ q j thì khi i ≥ j + 1
ta có x ≥ q i . Vậy nên

Xét hiệu

x − qi
x − qi
.
=
x + qi
x + qi

2q i (x − 1)
x − qi 1 − qi

=
≤ 0.

x + qi 1 + qi
(x + q i ) (1 + q i )

Do đó

p

p

k=j+1

x − qk
1 − qk
.

k
x + qk
1
+
q
k=j+1

Với k = 1, . . . , j, ta có

x − q j−(k−1)
q j−(k−1) − x
.
=
x + q j−(k−1)
q j−(k−1) + x

Ta lại có

2 q j+1 − x
q j−(k−1) − x 1 − q k

=
≤ 0.
q j−(k−1) − x 1 + q k
(1 + q k ) x + q j−(k−1)
Vậy nên
j

j

k=1

Từ đó ta được

p

k=1

1 − qk
x − qk

.
k
x + qk
1
+

q
k=1
p

x − qk
1 − qk

x + qk
1 + qk
k=1

hay

p

|f (x)| ≤
k=1

1 − qk
.
1 + qk


12

1.3

Định lý Viète

Định lý 1.2 (Định lý Viète (xem [6])). Giả sử f (x) ∈ R[x] có dạng


f (x) = an xn + an−1 xn−1 + · · · + a1 x + a0 , deg f = n.
Khi đó, nếu f có n nghiệm x1 , x2 , . . . , xn (có thể không phân biệt) thì

x1 + x2 + · · · + xn = −

an−1
,
an

x1 x2 + x1 x3 + · · · + xn−1 xn =

an−2
,
an

x1 x2 x3 + x1 x2 x4 + · · · + xn−2 xn−1 xn = −

an−3
,
an

...
x1 x2 . . . xn = (−1)n

a0
.
an

Chứng minh.

Do f (x) là đa thức bậc n và có n nghiệm x1 , x2 , . . . , xn nên

f (x) = an (x − x1 )(x − x2 ) . . . ..(x − xn )
= an xn − an (x1 + x2 + · · · + xn )xn−1 + · · · + (−1)n an x1 x2 . . . xn .
Dựa vào so sánh hệ số của f (x) theo cách khai triển trên và

f (x) = an xn + an−1 xn−1 + · · · + a1 x + a0 ,
ta được điều cần chứng minh.
Bài toán 1.12. Cho đa thức f (x) = x4 + a1 x3 + a2 x2 + a3 x + a4 có 4 nghiệm
không âm (có thể không phân biệt). Chứng minh

a3
4
Lời giải.

4

≥ a34 .


13

Gọi 4 nghiệm không âm của đa thức là x1 , x2 , x3 , x4 . Theo định lý Vi ét
ta có

x1 x2 x3 x4 = a4 ≥ 0,
x1 x2 x3 + x1 x2 x4 + x1 x3 x4 + x2 x3 x4 = −a3 ≥ 0.
Áp dụng bất đẳng thức AM - GM cho 4 số không âm ta có

−a3 = x1 x2 x3 + x1 x2 x4 + x1 x3 x4 + x2 x3 x4 ≥ 4 4 x31 x32 x33 x34 .

Suy ra

−a3
≥ 4 x31 x32 x33 x34 ,
4
a3
−a3
≥ 4 a34 ⇔
4
4
Đẳng thức xảy ra khi và chỉ khi

4

≥ a34 .

x1 x2 x3 = x1 x2 x4 = x1 x3 x4 = x2 x3 x4
⇔ x1 = x2 = x3 = x4 .
Có thể mở rộng Bài toán 1.12 cho đa thức bậc n như sau
Bài toán 1.13. Cho đa thức bậc n

f (x) = a0 xn + a1 xn−1 + · · · + an−1 x + an (a0 = 0)
có n nghiệm không âm (có thể không phân biệt). Chứng minh

an−1 n
≥ an−1
n .
n
Cách giải bài toán này cũng tương tự như trên.


1.4

Một số bất đẳng thức cơ bản

Trong mục này sẽ trình bày bất đẳng thức AM - GM, đây là một bất đẳng
thức quen thuộc. Từ bất đẳng thức AM - MG ta chứng minh được một số
bất đẳng thức dưới đây. Các bất đẳng thức này được áp dụng trong phần
sau của luận văn.


14

Bất đẳng thức AM - GM (xem[3])

1
n

n

1
n

n

xi ≥

xi

i=1


(1.6)

i=1

với n là số nguyên dương, xi là số thực không âm, ∀i = 1, n.

Bất đẳng thức 1.4.1 (xem[3])
1
n

n

1
n

n

≥1+

(1 + xi )

xi

(1.7)

i=1

i=1

với n là sô nguyên dương, xi là số thực dương, ∀i = 1, n.

Chứng minh.
Bất đẳng thức (1.7) tương đương với
n

1≥
i=1

1
1 + xi

1
n

n

xi
)
1 + xi

+
i=1

1
n

.

Áp dụng bất đẳng thức AM - GM ta có
n


i=1

1
1 + xi

1
n

n

+
i=1

xi
)
1 + xi

1
n

1

n
=

1
n

n


i=1
n

1
1
+
1 + xi n

n

i=1

xi
1 + xi

1
xi
+
1 + xi 1 + xi

i=1

= 1.

Đây là điều cần chứng minh.

Bất đẳng thức 1.4.2 (xem[3])

n


x2i + yi2 ≥
i=1

2

n

xi
i=1

2

n

+

yi
i=1

với n là sô nguyên dương, xi , yi là số thực dương, ∀i = 1, n.

(1.8)


15

Chứng minh.
Ta chứng minh bất đẳng thức (1.8) đúng với n = 2, tức là

x21 + y12 +


x22 + y22 ≥

(x1 + x2 )2 + (y1 + y2 )2 .

(1.9)

Thật vậy, bình phương hai vế của (1.9), ta có

(x21 + y12 )(x22 + y22 ) ≥ x1 x2 + y1 y2
⇔(x21 + y12 )(x22 + y22 ) ≥ (x1 x2 + y1 y2 )2
⇔(x1 y2 − x2 y1 )2 ≥ 0.
Vậy (1.9) đúng.
Giả sử bất đẳng thức (1.8) đúng với n = k , tức là
2

k

k

x2i + yi2 ≥

+

xi

yi

.


i=1

i=1

i=1

2

k

Ta cần chứng minh bất đẳng thức đúng với n = k + 1.
Xét

k+1

k

x2i

+

i=1

yi2

=

x2i + yi2 +

2 .

x2k+1 + yk+1

2

2

i=1

Suy ra
k+1

k

x2i + yi2 ≥
i=1

xi

k

+

yi

i=1

2
x2k+1 + yk+1

i=1

2

k+1



+

xi

2

k+1

+

yi

i=1

.

i=1

Vậy bất đẳng thức được chứng minh.

Bất đẳng thức 1.4.3 (xem[3])
n
n


i=1

x i yi

x i + yi

n

xi
i=1
n

yi
i=1
n

xi +
i=1

yi
i=1

(1.10)


16

với n là sô nguyên dương, xi , yi là số thực dương,∀i = 1, n.
Chứng minh.
Bất đẳng thức (1.10) tương đương với

n
n

i=1

xi yi
− yi
xi + yi


i=1

yi

i=1
n



i=1
n

n



yi

xi +


yi
i=1

i=1

i=1
2

n
n

n

xi

yi

yi2

xi + yi

i=1

.

n

n

yi


xi +
i=1

i=1

Ta có
2

n

=

yi
i=1
n


i=1

n

i=1
n

yi2
x i + yi


yi


xi + yi
xi + yi

2

n

xi +
i=1

yi .
i=1

Ta có điều cần chứng minh.

Bất đẳng thức 1.4.4 (xem[3])

1
n

n

xm
i ≥
i=1

1
n


m

n

xi

(1.11)

i=1

với m, n là số nguyên dương, xi là số thực không âm, ∀i = 1, n.
Chứng minh.
Ta chứng minh bất đẳng thức (1.11) bằng phương pháp qui nạp theo n.
Với n = 2 ta có (1.11) là
m
xm
1 + x2

2

x1 + x2
2

m

.


17


Ta có
m
xm
x1 + x2
xm−1
+ xm−1
1 + x2
1
2

2
2
2
m−1
m
xm
− x2 xm − 21
1 + x2 − x1 x2

≥0
4
⇔ x1m−1 − xm−1
(x1 − x2 ) ≥ 0.
2

Điều này đúng với mọi số thực dương x1 , x2 và m nguyên dương. Từ đó, ta
suy ra
m
+ xm−1
x1 + x2

xm−1
xm
1 + x2
2
1

2
2
2
2
x1 + x2
xm−2
+ xm−2
1
2

2
2
m
x1 + x2
.
≥ ··· ≥
2

Giả sử (1.11) đúng với n = k , ta sẽ chứng minh (1.11) đúng với n = 2k .
Thật vậy

1
2k


2k

xm
i
i=1

1

2

1
k

1
=
2

k

1
k

2k

xm
i

i=1
m


k

m

2k

1
xi
k i=k+1

+

xi

1
+
xm
k i=k+1 i

i=1



1
2k

m

2k


.

xi
i=1

Bất đẳng thức sẽ được chứng minh khi ta chứng minh được (1.11) đúng với

n = k + 1 thì sẽ đúng với n = k .
Thật vậy, ta cần chứng minh

1
k

k

xm
i ≥
i=1

hay
k

xm
i

P =
i=1

+


1
k

m

k

xi
i=1

m

k

xi
i=1

1
k

≥ (k + 1)

1
k

m

k

xi

i=1

.


18

Áp dụng giả thiết quy nạp, ta có

1
P ≥ (k + 1)
k+1

k

i=1

1
xi +
k

m

k

xi
i=1

Vậy bất đẳng thức (1.11) đã được chứng minh.


≥ (k + 1)

1
k

m

k

xi
i=1

.


19

Chương 2
Các dạng toán về đa thức và phân
thức với hệ số nguyên
2.1

Các dạng toán về đa thức một biến với hệ số
nguyên và hệ thức Viète

Trong mục này, sẽ trình bày các bài toán ước lượng một số đại lượng liên
quan đến đa thức hệ số nguyên như: ước lượng hệ số, ước lượng nghiệm, ước
lượng miền giá trị của đa thức,. . .
Bài toán 2.1. Cho tam thức bậc hai f (x) = ax2 + 1998x + c với a, c ∈ Z,


|a| < 2000, |c| < 2000 và f (x) có hai nghiệm phân biệt x1 , x2 . Chứng minh
|x1 − x2 | ≥

1
.
998

Lời giải.

f (x) có biệt thức ∆ = 9992 − ac, do f (x) có hai nghiệm phân biệt x1 , x2
nên ∆ > 0, ∆ ∈ Z và


2 ∆
|x1 − x2 | =
.
|a|

Nếu ∆ ≥ 2 thì




2 ∆
2 ∆
2 2
1
|x1 − x2 | =
>


>
|a|
2000
2000 998


20

(do |a| < 2000).
Nếu ∆ = 1 thì

ac = 9992 − 1 = 998.1000 = 24 .53 .449.
Do đó a là một ước số của 24 .53 .449, lại có 22 .449 = 1996 là một ước số của

24 .53 .449 và |a| < 2000 nên a ≤ 1996.
Từ đây ta suy ra


2
2
1
2 ∆
=

=
.
|x1 − x2 | =
|a|
|a| 1996 998


Bài toán 2.2. Cho đa thức P (x) = xn + an−1 xn−1 + · · · + a1 x + a0 và

|ai | = 1(i = 0, 1, . . . , n − 1). Chứng minh rằng nếu P (x) có n nghiệm thực
thì n ≤ 3.
Lời giải.
Giả sử P (x) có n nghiệm là x1 , x2 , . . . , xn . Áp dụng định lý Viète, ta có
n

xi = −an−1 ,
i=1
n

xi xj = an−2 .
i,j=1,i
Suy ra
2

xi

xi =
i=1

2

n

n

i=1


n

−2

xi xj = 3 (do|an−1 | = 1, |an−2 | = 1). (2.1)
i,j=1,i
Do x1 x2 . . . xn = (−1)n a0 và |a0 | = 1 nên xi = 0 ∀i.
1
là nghiệm của đa thức
Dễ thấy
xi
Q(x) = a0 xn + a1 xn−1 + · · · + an−1 x + 1.
Áp dụng Định lý Viète, ta có
n

−a1
1
=
,
xi
a0

i=1
n

1
a2
= .

xx
a0
i,j=1,i

21

Suy ra
n

i=1

1
=
xi 2

n

i=1

1
xi

2

n

−2

1

= 3, ( do |a1 | = 1, |a2 | = 1).
x
x
i
j
i,j=1,i
(2.2)

Từ (2.1) và (2.2), kết hợp với bất đẳng thức Cauchy, ta được
n

9=

n

xi

2

i=1

i=1

1
≥ n2 .
2
xi

Suy ra n ≤ 3.

Bài toán 2.3 (xem[5]). Cho đa thức P (x) có bậc m > 0 và có các hệ
số nguyên. Gọi n là số các nghiệm nguyên phân biệt của hai phương trình

P (x) = 1 và P (x) = −1. Chứng minh rằng n ≤ m + 2.
Lời giải.
Xét hai đa thức A(x) và B(x) với các hệ số nguyên

A(x) = an xn + an−1 xn−1 + · · · + a1 x + a0 ,
B(x) = an xn + an−1 xn−1 + · · · + a1 x + a0 + 2.
Gọi r, s là các nghiệm nguyên tương ứng của hai đa thức trên, tức là A(r) =

0, B(s) = 0. Ta có
n

ai (ri − si ) − 2 = 0.

A(r) − B(s) =
i=1

Từ đây suy ra nếu r − s = 0 thì 2 chia hết cho r − s, tức là chỉ xảy ra
một trong ba trường hợp r và s hơn kém nhau 0, 1 hoặc 2 đơn vị. Ta sẽ áp
dụng nhận xét trên để giải bài toán.
Giả sử r là nghiệm nguyên nhỏ nhất trong tất cả các nghiệm nguyên của
hai phương trình P (x) = 1 và P (x) = −1. Do đa thức P (x) có bậc m nên
mỗi phương trình trên không có quá m nghiệm.
Theo nhận xét trên nếu r là nghiệm của phương trình này, s là nghiệm của
phương trình kia và r là nghiệm nguyên nhỏ nhất trong tất cả các nghiệm
nguyên của hai phương trình thì s = r, s = r + 1 hoặc s = r + 2.



22

Do vậy nếu k là số nghiệm của phương trình thứ nhất thì số nghiệm của
cả hai phương trình nhiều nhất là k + 2 mà k ≤ m nên ta suy ra n ≤ m + 2.
Bài toán 2.4 (xem[5]). Cho f (x) là đa thức bậc n có các hệ số bằng ±1.
Biết rằng đa thức có x = 1 là nghiệm bội cấp m với m ≥ 2k , k ≥ 2 và k
nguyên. Chứng minh n ≥ 2k+1 − 1.
Lời giải.
Gọi f (x) là đa thức với các hệ số theo modulo 2 của f (x). Vì f (x) có các
hệ số là 1 và -1 nên

f (x) = xn + xn−1 + · · · + x + 1.
k

Ta có f (x) = (x − 1)2 g(x), trong đó n ≥ 2k và g(x) là đa thức hệ số
nguyên.
Dễ dàng chứng minh được C2i k ≡ 0(mod2), 1 ≤ i ≤ 2k − 1 nên
k

f (x) = xn + xn−1 + · · · + x + 1 = (x2 + 1)g(x).
Giả sử g(x) có bậc không quá 2k − 2. Ta có hệ số của x2
(2.3) là 0. Điều này mâu thuẫn vì hệ số của x2

k−1

k−1

(2.3)
ở vế phải của


ở vế trái của (2.3) là 1. Do

đó bậc của g(x) không nhỏ hơn 2k − 1. Vậy n ≥ 2k + 2k − 1 = 2k+1 − 1.
Bài toán 2.5. Cho đa thức với hệ số nguyên

P (x) = ak xk + · · · + a1 x + a0 .
Ta ký hiệu số các hệ số lẻ của đa thức là h(P ). Với i = 0, 1, 2, . . . ta đặt

Qi (x) = (1 + x)i . Chứng minh rằng nếu i1 , i2 , . . . , in là các số nguyên thỏa
mãn điều kiện 0 ≤ i1 < i2 < · · · < in thì ta có

h(Qi1 + Qi2 + · · · + Qin ) ≥ h(Qi1 ).
Lời giải.
Trước hết ta có nhận xét: Nếu i là một bội số của 2 thì tất cả các hệ số
của Qi đều chẵn, ngoại trừ hệ số đầu tiên và hệ số cuối cùng.


×